Saltar al contenido principal
LibreTexts Español

4.4: Regla de L'Hopital

  • Page ID
    107805
  • \( \newcommand{\vecs}[1]{\overset { \scriptstyle \rightharpoonup} {\mathbf{#1}} } \) \( \newcommand{\vecd}[1]{\overset{-\!-\!\rightharpoonup}{\vphantom{a}\smash {#1}}} \)\(\newcommand{\id}{\mathrm{id}}\) \( \newcommand{\Span}{\mathrm{span}}\) \( \newcommand{\kernel}{\mathrm{null}\,}\) \( \newcommand{\range}{\mathrm{range}\,}\) \( \newcommand{\RealPart}{\mathrm{Re}}\) \( \newcommand{\ImaginaryPart}{\mathrm{Im}}\) \( \newcommand{\Argument}{\mathrm{Arg}}\) \( \newcommand{\norm}[1]{\| #1 \|}\) \( \newcommand{\inner}[2]{\langle #1, #2 \rangle}\) \( \newcommand{\Span}{\mathrm{span}}\) \(\newcommand{\id}{\mathrm{id}}\) \( \newcommand{\Span}{\mathrm{span}}\) \( \newcommand{\kernel}{\mathrm{null}\,}\) \( \newcommand{\range}{\mathrm{range}\,}\) \( \newcommand{\RealPart}{\mathrm{Re}}\) \( \newcommand{\ImaginaryPart}{\mathrm{Im}}\) \( \newcommand{\Argument}{\mathrm{Arg}}\) \( \newcommand{\norm}[1]{\| #1 \|}\) \( \newcommand{\inner}[2]{\langle #1, #2 \rangle}\) \( \newcommand{\Span}{\mathrm{span}}\)\(\newcommand{\AA}{\unicode[.8,0]{x212B}}\)

    Ahora probamos un resultado que nos permite calcular varios límites calculando un límite relacionado que involucra derivados. Los cuatro teoremas de esta sección se conocen como Regla de L'Hopital.

    Para esta sección, asumimos\(a,b \in \mathbb{R}\) con\(a < b\).

    Teorema\(\PageIndex{1}\)

    Supongamos\(f\) y\(g\) son continuos\([a,b]\) y diferenciables en\((a,b)\). Supongamos\(f(\bar{x})=g(\bar{x})=0\), dónde\(\bar{x} \in [a,b]\). Supongamos además que existe\(\delta > 0\) tal que\(g^{\prime}(x) \neq 0\) para todos\(x \in B(\bar{x} ; \delta) \cap[a, b]\),\(x \neq \bar{x}\).

    Si

    \[\displaystyle \lim _{x \rightarrow \bar{x}} \frac{f^{\prime}(x)}{g^{\prime}(x)}=\ell,\]

    entonces

    \[\displaystyle \lim _{x \rightarrow \bar{x}} \frac{f(x)}{g(x)}=\ell.\]

    Prueba

    Que\(\left\{x_{k}\right\}\) sea una secuencia en la\([a,b]\) que converja hacia\(\bar{x}\) y tal que\(x_{k} \neq \bar{x}\) para cada uno\(k\). Por Teorema 4.2.4, para cada uno\(k\), existe una secuencia\(\left\{c_{k}\right\}\) entre\(c_{k}\) entre\(x_{k}\) y\(\bar{x}\), tal que

    \[\left[f\left(x_{k}\right)-f(\bar{x})\right] g^{\prime}\left(c_{k}\right)=\left[g\left(x_{k}\right)-g(\bar{x})\right] f^{\prime}\left(c_{k}\right).\]

    Ya que\(f(\bar{x})=g(\bar{x})=0\), y\(g^{\prime}\left(c_{k}\right) \neq 0\) para lo suficientemente grande\(k\), tenemos

    \[\frac{f\left(x_{k}\right)}{g\left(x_{k}\right)}=\frac{f^{\prime}\left(c_{k}\right)}{g^{\prime}\left(c_{k}\right)}.\]

    Bajo los supuestos que\(g^{\prime}(x) \neq 0\) para\(x\) cerca\(\bar{x}\) y\(g(\bar{x})=0\), también tenemos\(g\left(x_{k}\right) \neq 0\) para suficientemente grandes\(k\). Por el teorema squeeze (Teorema 2.1.6),\(\left\{c_{k}\right\}\) converge a\(\bar{x}\). Así,

    \[\displaystyle \lim _{k \rightarrow \infty} \frac{f\left(x_{k}\right)}{g\left(x_{k}\right)}=\displaystyle \lim _{k \rightarrow \infty} \frac{f^{\prime}\left(c_{k}\right)}{g^{\prime}\left(c_{k}\right)}=\displaystyle \lim _{x \rightarrow \bar{x}} \frac{f^{\prime}(x)}{g^{\prime}(x)}=\ell .\]

    Por lo tanto, (4.9) se desprende del Teorema 3.1.2. \(\square\)

    Ejemplo\(\PageIndex{1}\)

    Utilizaremos el Teorema 4.4.1 para demostrar que\[\displaystyle \lim _{x \rightarrow 0} \frac{2 x+\sin x}{x^{2}+3 x}=1. \nonumber\]

    Solución

    Primero observamos que se mantienen las condiciones del Teorema 4.4.1. Aquí\(f(x)=2 x+\sin x\),\(g(x)=x^{2}+3 x\), y\(\bar{x} = 0\). Podemos tomar\([a, b]=[-1,1]\), por ejemplo, para que\(f\) y\(g\) sean continuos\([a, b]\) y diferenciables\((a, b)\) y, además,\(\frac{f(x)}{g(x)}\) estén bien definidos en\([a, b] \backslash \{\bar{x}\}\). Además, tomando\(\delta = 7 / 3\), obtenemos\(g^{\prime}(x)=2 x+3 \neq 0\) para\(x \in B(\bar{x} ; \delta) \cap[a, b]\). Finalmente calculamos el límite del cociente de derivados usando el Teorema 3.2.1 para obtener

    \[\begin{align*} \displaystyle \lim _{x \rightarrow \bar{x}} \frac{f^{\prime}(x)}{g^{\prime}(x)} & =\displaystyle \lim _{x \rightarrow 0} \frac{2+\cos x}{2 x+3} \\[4pt] &=\frac{\displaystyle \lim _{x \rightarrow 0} 2+\displaystyle \lim _{x \rightarrow 0} \cos x}{\displaystyle \lim _{x \rightarrow 0} 2 x+3} \\[4pt] &=\frac{2+1}{3} \\[4pt] &=1.\end{align*}\]

    Ahora se deduce del Teorema 4.4.1 que\(\displaystyle \lim _{x \rightarrow 0} \frac{2 x+\sin x}{x^{2}+3 x}=1\) como queríamos mostrar.

    Ejemplo\(\PageIndex{2}\)

    Aplicaremos la regla de L'Hospital para determinar el límite\[\displaystyle \lim _{x \rightarrow 1} \frac{3 x^{3}-2 x^{2}+4 x-5}{4 x^{4}-2 x-2}.\nonumber\]

    Solución

    Aquí\(f(x)=3 x^{3}-2 x^{2}+4 x-5\) y\(g(x)=4 x^{4}-2 x-2\). Así\(f(1)=g(1)=0\). Por otra parte,\(f^{\prime}(x)= 9 x^{2}-4 x+4\) y\(g^{\prime}(x)=16 x^{3}-2\). Ya que\(g^{\prime}(1)=14 \neq 0\) y\(g^{\prime}\) es continuo tenemos\(g^{\prime}(x) \neq 0\) por\(x\) cerca\(1\). Ahora,

    \[\displaystyle \lim _{x \rightarrow 1} \frac{9 x^{2}-4 x+4}{16 x^{3}-2}=\frac{9}{14}.\nonumber\]

    Así, el límite deseado también\(\frac{9}{14}\) lo es.

    Ejemplo\(\PageIndex{3}\)

    Si las derivadas de las funciones\(f\) y\(g\) ellas mismas satisfacen los supuestos del Teorema 4.4.1 podremos aplicar la regla de L'Hospital para determinar primero el límite de\(f^{\prime}(x) / g^{\prime}(x)\) y luego aplicar nuevamente la regla para determinar el límite original.

    Solución

    Considera el límite

    \[\displaystyle \lim _{x \rightarrow 0} \frac{x^{2}}{1-\cos x}.\nonumber\]

    Aquí\(f(x)=x^{2}\) y\(g(x)=1-\cos x\) así ambas funciones y todas sus derivadas son continuas. Ahora\(g^{\prime}(x)=\sin x\) y, entonces,\(g^{\prime}(x) \neq 0\) para\(x\) cerca de cero,\(x \neq 0\). También\(f^{\prime}(0)=0=g^{\prime}(0)\) y\(g^{\prime \prime}(x)=\cos x \neq 0\) para\(x\) cerca\(0\). Además,

    \[\displaystyle \lim _{x \rightarrow 0} \frac{f^{\prime \prime}(x)}{g^{\prime \prime}(x)}=\displaystyle \lim _{x \rightarrow 0} \frac{2}{\cos x}=2 .\nonumber\]

    Por regla de L'Hospital obtenemos

    \[\displaystyle \lim _{x \rightarrow 0} \frac{f^{\prime}(x)}{g^{\prime}(x)}=\displaystyle \lim _{x \rightarrow 0} \frac{f^{\prime \prime}(x)}{g^{\prime \prime}(x)}=\displaystyle \lim _{x \rightarrow 0} \frac{2}{\cos x}=2 .\nonumber\]

    Aplicando la regla de L'Hospital una vez más obtenemos

    \[\displaystyle \lim _{x \rightarrow 0} \frac{x^{2}}{1-\cos x}=\displaystyle \lim _{x \rightarrow 0} \frac{f(x)}{g(x)}=\displaystyle \lim _{x \rightarrow 0} \frac{f^{\prime}(x)}{g^{\prime}(x)}=2 .\nonumber\]

    Ejemplo\(\PageIndex{4}\)

    Dejar\(g(x)=x+3 x^{2}\) y dejar\(f: \mathbb{R} \rightarrow \mathbb{R}\) ser dado por

    \ [f (x) =\ left\ {\ begin {array} {ll}
    x^ {2}\ sin\ frac {1} {x}, &\ text {if} x\ neq 0\ text {;}\\
    0, &\ text {if} x=0.
    \ end {array}\ right. \ nonumber\]

    Solución

    Ahora considera el límite

    \[\displaystyle \lim _{x \rightarrow 0} \frac{f(x)}{g(x)}=\displaystyle \lim _{x \rightarrow 0} \frac{x^{2} \sin \frac{1}{x}}{x+3 x^{2}} . \nonumber\]

    Usando las reglas derivadas at\(x \neq 0\) y la definición de derivada en\(x = 0\) podemos ver que\(f\) es diferenciable y\ [f^ {\ prime} (x) =\ left\ {\ begin {array} {ll}
    2 x\ sin\ frac {1} {x} -\ cos\ frac {1} {x}, &\ text {if} x\ neq 0\ text {;}\\
    0, &\ text {if} x= 0\ text {,}
    \ end {array}\ right. \ nonumber\]

    Sin embargo, no\(f^{\prime}\) es conínua en\(0\) (ya que\(\displaystyle \lim _{x \rightarrow 0} f^{\prime}(x)\) no existe) y, por lo tanto, la regla de L'Hospital no se puede aplicar en este caso.

    Por otro lado\(\displaystyle \lim _{x \rightarrow 0} \frac{x^{2} \sin \frac{1}{x}}{x+3 x^{2}}\) sí existe como podemos ver en

    \[\displaystyle \lim _{x \rightarrow 0} \frac{x^{2} \sin \frac{1}{x}}{x+3 x^{2}}=\displaystyle \lim _{x \rightarrow 0} \frac{x \sin \frac{1}{x}}{1+3 x}=\frac{\displaystyle \lim _{x \rightarrow 0} x \sin \frac{1}{x}}{\displaystyle \lim _{x \rightarrow 0}(1+3 x)}=0 . \nonumber\]

    Teorema\(\PageIndex{2}\)

    Vamos\(a, b \in \mathbb{R}\),\(a < b\), y\(\bar{x} (a, b)\). Supongamos que\(f, g:(a, b) \backslash\{\bar{x}\} \rightarrow \mathbb{R}\) son diferenciables\((a, b) \backslash\{\bar{x}\}\) y asumen\(\displaystyle \lim _{x \rightarrow \bar{x}} f(x)=\displaystyle \lim _{x \rightarrow \bar{x}} g(x)=\infty\). Supongamos además que existe\(\delta > 0\) tal que\(g^{\prime}(x) \neq 0\) para todos\(x \in B(\bar{x} ; \delta) \cap(a, b)\),\(x \neq \bar{x}\).

    Si\(\ell \in \mathbb{R}\) y

    \[\displaystyle \lim _{x \rightarrow \bar{x}} \frac{f^{\prime}(x)}{g^{\prime}(x)}=\ell ,\]

    entonces

    \[\displaystyle \lim _{x \rightarrow \bar{x}} \frac{f(x)}{g(x)}=\ell .\]

    Prueba

    Ya que\(\displaystyle \lim _{x \rightarrow \bar{x}} f(x)=\displaystyle \lim _{x \rightarrow \bar{x}} g(x)=\infty\), eligiendo un positivo más pequeño\(\delta\) si es necesario, podemos asumir eso\(f(x) \neq 0\) y\(g(x) \neq 0\) para todos\(x \in B(\bar{x} ; \delta) \cap(a, b)\).

    Eso lo demostraremos\(\displaystyle \lim _{x \rightarrow \bar{x}^{+}} \frac{f(x)}{g(x)}=\ell\). La prueba que\(\displaystyle \lim _{x \rightarrow \bar{x}^{-}} \frac{f(x)}{g(x)}=\ell\) es completamente análoga.

    Arreglar cualquier\(\varepsilon > 0\). Tenemos que encontrar\(\delta_{0}>0\) tal que\(|f(x) / g(x)-\ell|<\varepsilon\) cuando sea\(x \in B_{+}\left(\bar{x} ; \delta_{0}\right) \cap (a, b)\).

    De\(\PageIndex{18}\), uno cn elegir\(K > 0\) y un positivo\(\delta_{1}<\delta\) tal que\[\left|\frac{f^{\prime}(x)}{g^{\prime}(x)}\right| \leq K \text { and }\left|\frac{f^{\prime}(x)}{g^{\prime}(x)}-\ell\right|<\frac{\varepsilon}{2}\] siempre que\(x \in B\left(\bar{x} ; \delta_{1}\right) \cap(a, b)\),\(x \neq \bar{x}\).

    Fijar\(\alpha \in B_{+}\left(\bar{x} ; \delta_{1}\right) \cap(a, b)\) (en particular,\(\alpha > \bar{x}\). Ya que\(\displaystyle \lim _{x \rightarrow \bar{x}} f(x)=\infty\), podemos encontrar\(\delta_{2}>0\) tal que\(\delta_{2}<\min \left\{\delta_{1}, \alpha-\bar{x}\right\}\) y\(f(x) \neq f(\alpha)\) para\(x \in B_{+}\left(\bar{x} ; \delta_{2}\right) \cap(a, b)=B_{+}\left(\bar{x} ; \delta_{2}\right)\). Además, para tal\(x\), ya que\(g^{\prime}(z) \neq 0\) si\(x < z < \alpha\), el teorema de Rolle (Teorema 4.2.2) lo garantiza\(g(x) \neq g(\alpha)\). Por lo tanto, para todos\(x \in B_{+}\left(\bar{x} ; \delta_{2}\right)\) podemos escribir,

    \[\frac{f(x)}{g(x)}=\frac{f(x)-f(\alpha)}{g(x)-g(\alpha)} \frac{1-\frac{g(\alpha)}{g(x)}}{1-\frac{f(\alpha)}{f(x)}} .\]

    Ahora, defina

    \[H_{\alpha}(x)=\frac{1-\frac{g(\alpha)}{g(x)}}{1-\frac{f(\alpha)}{f(x)}} \text { for } x \in B_{+}\left(\bar{x} ; \delta_{2}\right) .\]Ya que\(\displaystyle \lim _{x \rightarrow \bar{x}} f(x)=\displaystyle \lim _{x \rightarrow \bar{x}} g(x)=\infty\), tenemos eso\(\displaystyle \lim _{x \rightarrow \bar{x}^{+}} H_{\alpha}(x)=1\). Así, existe un positivo\(\gamma<\delta_{2}\) tal que

    \[\left|H_{\alpha}(x)-1\right|<\frac{\varepsilon}{2 K} \text { whenever } x \in B_{+}(\bar{x} ; \gamma) .\]

    Para cualquiera\(x \in B_{+}(\bar{x} ; \gamma)\), aplicando el Teorema 4.2.4 sobre el intervalo\([x, \alpha]\), podemos escribir\([f(x)-f(\alpha)] g^{\prime}(c)= [g(x)-g(\alpha)] f^{\prime}(c)\) para algunos\(c \in (x, \alpha)\) (nótese que, en particular,\(c \in B\left(\bar{x}: \delta_{1}\right) \cap(a, b)\)). Por tal\(c\) obtenemos\[\frac{f(x)}{g(x)}=\frac{f^{\prime}(c)}{g^{\prime}(c)} H_{\alpha}(x) .\] Desde\(c \in B\left(\bar{x}: \delta_{1}\right) \cap(a, b)\), aplicando\(\PageIndex{20}\) obtenemos que, para\(x \in B_{+}(\bar{x} ; \gamma)=B_{+}(\bar{x} ; \gamma) \cap(a, b)\),

    \ [\ begin {alineado}
    \ izquierda|\ frac {f (x)} {g (x)} -\ ell\ derecha| &=\ izquierda|\ frac {f^ {\ prime} (c)} {g^ {\ prime} (c)} H_ {\ alpha} (x) -\ ell\ derecha|\\
    &=\ izquierda|\ frac {f^ {prime} (c)} {g^ {\ prime} (c)}\ izquierda (H_ {\ alfa} (x) -1\ derecha) +\ frac {f^ {\ prime} (c)} {g^ {\ prime} (c)} -\ ell\ derecha|\\
    &\ leq\ izquierda|\ frac {f^ {\ prime} (c)} {g^ {\ prime} (c)}\ derecha|\ izquierda|h_ {\ alpha} (x) -1\ derecha|+\ izquierda|\ frac {f^ {\ prime} (c)} {g^ {\ prime} (c)} -\ ell\ derecha|\
    &<K\ frac {\ varepsilon} {2 K} +\ frac {\ varepsilon} {2} =\ varepsilon\ texto {.}
    \ end {alineado}\]

    El ajuste\(\delta_{0}=\gamma\) completa la prueba. \(\square\)

    Ejemplo\(\PageIndex{5}\)

    Considera el límite\[\displaystyle \lim _{x \rightarrow 0} \frac{\ln x^{2}}{1+\frac{1}{\sqrt[3]{x^{2}}}} . \nonumber\]

    Solución

    Aquí\(f(x)=\ln x^{2}\),\(g(x)=1+\frac{1}{\sqrt[3]{x^{2}}}\),\(\bar{x} = 0\), y podemos tomar como\((a, b)\) cualquier intervalo abierto que contenga\(0\). Claramente\(f\) y\(g\) satisfacer los supuestos de diferenciabilidad y\(g^{\prime}(x) \neq 0\) para todos\(x \neq 0\). Por otra parte,\(\displaystyle \lim _{x \rightarrow \bar{x}} f(x)=\displaystyle \lim _{x \rightarrow \bar{x}} g(x)=\infty\). Analizamos el cociente de los derivados. Tenemos

    \[\displaystyle \lim _{x \rightarrow 0} \frac{2 / x}{-\frac{2}{3} \frac{1}{\sqrt[3]{x^{5}}}}=\displaystyle \lim _{x \rightarrow 0}-3 \frac{\sqrt[3]{x^{5}}}{x}=\displaystyle \lim _{x \rightarrow 0}-3 \sqrt[3]{x^{2}}=0 . \nonumber\]

    Ahora se deduce del Teorema 4.4.2 que

    \[\displaystyle \lim _{x \rightarrow 0} \frac{\ln x^{2}}{1+\frac{1}{\sqrt[3]{x^{2}}}}=0 . \nonumber\]

    OBSERVACIÓN\(\PageIndex{3}\)

    Las pruebas del Teorema 4.4.1 y del Teorema 4.4.2 muestran que los resultados en estos teoremas se pueden aplicar para los límites de mano izquierda y derecha. Además, los resultados también se pueden modificar para incluir el caso cuando\(\bar{x}\) es un punto final del dominio de las funciones\(f\) y\(g\).

    El siguiente teorema se puede probar siguiendo el método en la prueba del Teorema 4.4.1.

    Teorema\(\PageIndex{4}\)

    Dejar\(f\) y\(g\) ser diferenciable en\((a, \infty\). Supongamos\(g^{\prime}(x) \neq 0\) para todos\(x \in (a, \infty\) y

    \[\displaystyle \lim _{x \rightarrow \infty} f(x)=\displaystyle \lim _{x \rightarrow \infty} g(x)=0 .\]

    Si\(\ell \in \mathbb{R}\) y

    \[\displaystyle \lim _{x \rightarrow \infty} \frac{f^{\prime}(x)}{g^{\prime}(x)}=\ell ,\]entonces\[\displaystyle \lim _{x \rightarrow \infty} \frac{f(x)}{g(x)}=\ell .\]

    Ejemplo\(\PageIndex{6}\)

    Considera el límite\[\displaystyle \lim _{x \rightarrow \infty} \frac{1}{x\left(\frac{\pi}{2}-\arctan x\right)} . \nonumber\]

    Solución

    Escribiendo el cociente en la forma\(\frac{1 / x}{\frac{\pi}{2}-\arctan x}\) podemos aplicar Teorema 4.4.4. Ahora calculamos el límite del cociente de los derivados

    \[\displaystyle \lim _{x \rightarrow \infty} \frac{-1 / x^{2}}{-\frac{1}{x^{2}+1}}=\displaystyle \lim _{x \rightarrow \infty} \frac{x^{2}+1}{x^{2}}=1 . \nonumber\]

    En vista del Teorema 4.4.4 el límite deseado es también\(1\).

    El siguiente teorema se puede probar siguiendo el método en la prueba del teorema 4.4.2.

    Teorema\(\PageIndex{5}\)

    Dejar\(f\) y\(g\) ser diferenciable en\((a, \infty)\). Supongamos\(g^{\prime}(x) \neq 0\) para todos\(x \in (a, \infty)\) y\[\displaystyle \lim _{x \rightarrow \infty} f(x)=\displaystyle \lim _{x \rightarrow \infty} g(x)=\infty . \] si\(\ell \in \mathbb{R}\) y\[\displaystyle \lim _{x \rightarrow \infty} \frac{f^{\prime}(x)}{g^{\prime}(x)}=\ell ,\] luego\[\displaystyle \lim _{x \rightarrow \infty} \frac{f(x)}{g(x)}=\ell .\]

    Ejemplo\(\PageIndex{7}\)

    Considera el límite\[\displaystyle \lim _{x \rightarrow \infty} \frac{\ln x}{x} .\]

    Solución

    Claramente las funciones\(f(x)=\ln x\) y\(g(x)=x\) satisfacen las condiciones del Teorema 4.4.5. Tenemos

    \[\displaystyle \lim _{x \rightarrow \infty} \frac{f^{\prime}(x)}{g^{\prime}(x)}=\displaystyle \lim _{x \rightarrow \infty} \frac{1 / x}{1}=0 \nonumber\]

    Del Teorema 4.4.5 se desprende que\(\displaystyle \lim _{x \rightarrow \infty} \frac{\ln x}{x}=0\)

    Ejercicio\(\PageIndex{1}\)

    Utilice la Regla de L'Hospital para encontrar los siguientes límites (puede suponer que conoce todas las derivadas relevantes del cálculo):

    1. \(\displaystyle \lim _{x \rightarrow-2} \frac{x^{3}-4 x}{3 x^{2}+5 x-2}\).
    2. \(\displaystyle \lim _{x \rightarrow 0} \frac{e^{x}-e^{-x}}{\sin x \cos x}\).
    3. \(\displaystyle \lim _{x \rightarrow 1} \frac{x-1}{\sqrt{x+1}-\sqrt{2}}\).
    4. \(\displaystyle \lim _{x \rightarrow 0} \frac{e^{x}-e^{-x}}{\ln (1+x)}\).
    5. \(\displaystyle \lim _{x \rightarrow 1} \frac{\ln x}{\sin (\pi x)}\).

    Ejercicio\(\PageIndex{2}\)

    Para los problemas a continuación use la regla de L'Hospital tantas veces como corresponda para determinar los limts.

    1. \(\displaystyle \lim _{x \rightarrow 0} \frac{1-\cos 2 x}{x \sin x}\).
    2. \(\displaystyle \lim _{x \rightarrow 0} \frac{\left(x-\frac{\pi}{2}\right)^{2}}{1-\sin x}\).
    3. \(\displaystyle \lim _{x \rightarrow 0} \frac{x-\arctan x}{x^{3}}\).
    4. \(\displaystyle \lim _{x \rightarrow 0} \frac{x-\sin x}{x-\tan x}\).

    Ejercicio\(\PageIndex{3}\)

    Utilice la versión relevante de la regla de L'Hospital para calcular cada uno de los siguientes límites.

    1. \(\displaystyle \lim _{x \rightarrow \infty} \frac{3 x^{2}+2 x+7}{4 x^{2}-6 x+1}\).
    2. \(\displaystyle \lim _{x \rightarrow 0^{+}} \frac{-\ln x}{\cot x}\).
    3. \(\displaystyle \lim _{x \rightarrow \infty} \frac{\frac{\pi}{2}-\arctan x}{\ln \left(1+\frac{1}{x}\right)}\).
    4. \(\displaystyle \lim _{x \rightarrow \infty} \sqrt{x} e^{-x}\). (Pista: primero reescribir como cociente.)

    Ejercicio\(\PageIndex{4}\)

    Demostrar que las siguientes funciones son diferenciables en\(1\) y\(-1\).

    1. \ (f (x) =\ left\ {\ begin {array} {ll}
      x^ {2} e^ {-x^ {2}}, &\ text {if} |x|\ leq 1\ text {;}\
      \ frac {1} {e}, &\ text {if} |x|>1\ text {.}
      \ end {array}\ derecho.\)
    2. \ (f (x) =\ left\ {\ begin {array} {ll}
      \ arctan x, &\ text {if} |x|\ leq 1\ text {;}\\
      \ frac {\ pi} {4}\ nombreoperador {signo} x+\ frac {x-1} {2}, &\ text {if} |x|>1\ texto {.}
      \ end {array}\ derecho.\)

    Ejercicio\(\PageIndex{5}\)

    \(P(x)\)Déjese ser un polinomio. Demostrar que\[\displaystyle \lim _{x \rightarrow \infty} P(x) e^{-x}=0 . \nonumber\]

    Ejercicio\(\PageIndex{6}\)

    Considere la función

    \ [f (x) =\ left\ {\ begin {array} {ll}
    e^ {-\ frac {1} {x^ {2}}}, &\ text {if} x\ neq 0\ text {;}\\
    0, &\ text {if} x=0\ text {.}
    \ end {array}\ derecho.\]

    \(f \in C^{n}(\mathbb{R})\)Demuéstralo para cada\(n \in \mathbb{N}\).


    This page titled 4.4: Regla de L'Hopital is shared under a CC BY-NC-SA license and was authored, remixed, and/or curated by Lafferriere, Lafferriere, and Nguyen (PDXOpen: Open Educational Resources) .